1953 AHSME Problems/Problem 2

Revision as of 20:49, 16 January 2019 by Brendanb4321 (talk | contribs) (Solution)
(diff) ← Older revision | Latest revision (diff) | Newer revision → (diff)

Problem

A refrigerator is offered at sale at $250.00 less successive discounts of 20% and 15%. The sale price of the refrigerator is:

$\textbf{(A) } \text{35\% less than 250.00} \qquad \textbf{(B) } \text{65\% of 250.00} \qquad \textbf{(C) } \text{77\% of 250.00} \qquad  \textbf{(D) } \text{68\% of 250.00} \qquad \textbf{(E) } \text{none of these}$

Solution

The first discount takes off $20\%$ of the price, so the cost of the refrigerator is $0.8\cdot250$. The next discount takes off $15\%$, so the cost of the refrigerator is now $0.8\cdot0.85\cdot250=0.68\cdot250$. Thus, the sale price of the refrigerator is $\boxed{\textbf{(D)}\ \text{68\% of 250.00}}$.

See Also

1953 AHSC (ProblemsAnswer KeyResources)
Preceded by
Problem 1
Followed by
Problem 3
1 2 3 4 5 6 7 8 9 10 11 12 13 14 15 16 17 18 19 20 21 22 23 24 25 26 27 28 29 30 31 32 33 34 35 36 37 38 39 40 41 42 43 44 45 46 47 48 49 50
All AHSME Problems and Solutions

The problems on this page are copyrighted by the Mathematical Association of America's American Mathematics Competitions. AMC logo.png